Đến nội dung

angleofdarkness nội dung

Có 245 mục bởi angleofdarkness (Tìm giới hạn từ 29-04-2020)



Sắp theo                Sắp xếp  

#498204 $A = (\frac{6x+4}{3\sqrt{3x^{3}}-8}-\frac{\sqrt{3x}}...

Đã gửi bởi angleofdarkness on 10-05-2014 - 12:35 trong Đại số

Cho biết thức : $A = (\frac{6x+4}{3\sqrt{3x^{3}}-8}-\frac{\sqrt{3x}}{3x+2\sqrt{3x}+4})(\frac{1+3\sqrt{3x^{3}}}{1+\sqrt{3x}}-\sqrt{3x})$

a)Rút gọn A
b)Tìm các giá trị nguyên của x để biểu thức A nhận giá trị nguyên

 

a/ ĐKXĐ: x khác $\frac{4}{3}$; x > 0

 

$A = (\frac{6x+4}{3\sqrt{3x^{3}}-8}-\frac{\sqrt{3x}}{3x+2\sqrt{3x}+4})(\frac{1+3\sqrt{3x^{3}}}{1+\sqrt{3x}}-\sqrt{3x}) \\ =\frac{6x+4-\sqrt{3x}(\sqrt{3x}-2)}{(\sqrt{3x}-2)(3x+2\sqrt{3x}+4)}. \Big( \frac{(1+\sqrt{3x})(1-\sqrt{3x}+3x)}{1+\sqrt{3x}}-\sqrt{3x} \Big) \\ =\frac{3x+2\sqrt{3x}+4}{(\sqrt{3x}-2)(3x+2\sqrt{3x}+4)}. (1-\sqrt{3x}+3x-\sqrt{3x}) \\ =\frac{3x-2\sqrt{3x}+1}{\sqrt{3x}-2}$

 

b/ G/s A = m ( $m \in \mathbb{Z}$ ) $\Rightarrow 3x-(2+m)\sqrt{3x}+1+2m=0$ Do $x \in \mathbb{Z}$ nên xét:

 

- Xét $\sqrt{3x} \in \mathbb{I}$ mà $\sqrt{3x}=\dfrac{3x+1+2m}{2+m} \in \mathbb{Q}$ (vô lí)

 

- Xét $\sqrt{3x} \in \mathbb{Z}$ thì có:

$$A=\frac{3x-2\sqrt{3x}+1}{\sqrt{3x}-2}=\sqrt{3x}+\dfrac{1}{\sqrt{3x}-2}$$

Từ đó thấy $\sqrt{3x}-2$ là ước nguyên của 1.




#496502 Trận 8 - Hình học

Đã gửi bởi angleofdarkness on 01-05-2014 - 22:18 trong Thi giải toán Marathon cấp THCS 2014

Chỉ là nhầm lẫn nhưng mình nghĩ là không thể rồi @@ (Vì cái này ngang với ấn vào nút Sửa)
Nhưng bài kia vẫn được nửa số điểm đấy

 

Đc nửa điểm toàn bài là cùng thôi Hiếu.

 




#496079 Trận 6 - Phương trình nghiệm nguyên, đồng dư, chia hết

Đã gửi bởi angleofdarkness on 30-04-2014 - 13:43 trong Thi giải toán Marathon cấp THCS 2014

  Vì lý do cá nhân nên sẽ chấm trận 6 trước .Đã chấm xong bài các toán thủ. Các bạn có một ngày để phúc khảo điểm .

Còn bảng điểm cụ thể sẽ được lập sau 

 

 Bài em chưa có điểm ?




#495412 Điều lệ Các cuộc thi Marathon năm 2014

Đã gửi bởi angleofdarkness on 27-04-2014 - 11:08 trong Thi giải toán Marathon cấp THCS 2014

$t_{lb1}$: là thời điểm có toán thủ đầu tiên làm bài đúng. Cho em hỏi toán thủ đó là người ra đề thì có tính không ạ. 
Mong là không :)

 

Không, bài làm của toán thủ ra đề là đáp án chứ không phải bài tham gia như các toán thủ khác.




#495289 Topic về Bất đẳng thức, cực trị THCS

Đã gửi bởi angleofdarkness on 26-04-2014 - 19:25 trong Bất đẳng thức và cực trị

Cho x,y,z$\in [1;2]$. Tìm max

A=(x+y+z)($\frac{1}{x}+\frac{1}{y}+\frac{1}{z}$)

 

untitled38.png

 

Mình chụp hình lời giải (bàn phím lỗi k gửi đc bài -_- )

 

P/S: mình đổi nhầm biến x; y; z thành a; b; c :P Bạn thông cảm!




#495285 Topic về Bất đẳng thức, cực trị THCS

Đã gửi bởi angleofdarkness on 26-04-2014 - 18:59 trong Bất đẳng thức và cực trị

Cho x,y>0 và x+y=1.Cm 

3(3x-2)$^{2}$+ $\frac{8x}{y}$$\geq$7

 

 

Cho x,y,z$\in [1;2]$. Tìm max

A=(x+y+z)($\frac{1}{x}+\frac{1}{y}+\frac{1}{z}$)

 

Bài 1/

 

Bạn tham khảo các cách làm ở pic này




#495209 Topic về Bất đẳng thức, cực trị THCS

Đã gửi bởi angleofdarkness on 26-04-2014 - 12:05 trong Bất đẳng thức và cực trị


Câu 3:

Trên mathlinks.ro có bài toán sau:

“Cho các số thực dương a, b, c thỏa $3a+4b+6c\ge 42$. Chứng minh

$a+b+c+\frac{3}{a}+\frac{6}{b}+\frac{8}{c}\ge \frac{29}{2}$.

Lời giải trên trang ấy là

BĐT đề bài tương đương $\frac{3{{\left( a-2 \right)}^{2}}}{4a}+\frac{2{{\left( b-3 \right)}^{2}}}{3b}+\frac{{{\left( c-4 \right)}^{2}}}{2c}+\frac{3a+4b+6c-42}{12}\ge 0$ (1) ”

hãy chỉ mình phương pháp tìm ra (1).

Mình sẽ rất vui nếu như bạn trả lời các câu hỏi của mình. 

 

Chỉ là chuyển $\dfrac{29}{2}$ sang VT để có VT $\geq 0$ cho gọn và biến đổi tương đương thành tổng các phân số sao cho có phân số chứa tử là 3a + 4b + 6c - 42 (cái cụm này đã $\geq 0$)

 

Các số còn thừa lại nhóm thích hợp theo cụm chứa a, b, c để tạo HĐT là đc.

 




#495207 Topic về Bất đẳng thức, cực trị THCS

Đã gửi bởi angleofdarkness on 26-04-2014 - 11:59 trong Bất đẳng thức và cực trị

Câu 2

Trong cuốn “Sáng tạo bất đẳng thức” có một bài toán thế này

“Chứng minh với mọi số thực dương a, b, c ta có

$\frac{{{a}^{3}}b}{1+a{{b}^{2}}}+\frac{{{b}^{3}}c}{1+b{{c}^{2}}}+\frac{{{c}^{3}}a}{1+c{{a}^{2}}}\ge \frac{abc\left( a+b+c \right)}{1+abc}$.

Lời giải như sau:

Với mọi số thực dương k, áp dụng BĐT Cauchy - Schwarz ta có

$\frac{{{a}^{2}}}{b+kc}+\frac{{{b}^{2}}}{c+ka}+\frac{{{c}^{2}}}{a+kb}\ge \frac{a+b+c}{k+1}$

Ta chọn $k=\frac{1}{abc}$ thì có được đpcm.”

Mình xin hỏi tại sao lại nghĩ đến BĐT $\frac{{{a}^{2}}}{b+kc}+\frac{{{b}^{2}}}{c+ka}+\frac{{{c}^{2}}}{a+kb}\ge \frac{a+b+c}{k+1}$, và chọn $k=\frac{1}{abc}$.

 

 

 

Ta thấy tử là $a^3b$ và mẫu chứa $a^2b$; hai số này có nhân tử chung là ab nên chia cả tử và mẫu cho ab sẽ gọn hơn.

 

Như vậy ta sẽ có $\dfrac{{{a}^{3}}b}{1+a{{b}^{2}}}=\dfrac{a^2}{\dfrac{1}{ab}+b}$ 

 

Tương tự thì $\dfrac{{{b}^{3}}c}{1+b{{c}^{2}}}=\dfrac{b^2}{\dfrac{1}{bc}+c}$ và $\dfrac{{{c}^{3}}a}{1+c{{a}^{2}}}=\dfrac{c^2}{\dfrac{1}{ca}+a}$

 

Đến lúc này thì có thể dùng BĐT Cauchy - Schwarz rồi, mẫu là $\dfrac{1}{ab}+b +\dfrac{1}{bc}+c +\dfrac{1}{ca}+a$

 

Nhưng để cho gọn (nhóm mẫu mới vào sao cho có hạng tử a + b + c ) thì cần biến đổi $\dfrac{1}{ab}=\dfrac{c}{abc}$, các hạng tử này đã đồng nhất 

 

Xong.




#495205 Topic về Bất đẳng thức, cực trị THCS

Đã gửi bởi angleofdarkness on 26-04-2014 - 11:45 trong Bất đẳng thức và cực trị

Mình có một số vấn đề về BĐT xin bạn giúp đỡ.

Câu 1: Trong cuốn sách Phương pháp giải toán BĐT và cực trị dành cho học sinh 8, 9

Cho 3 số thực không âm a, b, c. Chứng minh rằng

${{\left( {{a}^{2}}+{{b}^{2}}+{{c}^{2}} \right)}^{2}}\ge 4\left( a+b+c \right)\left( a-b \right)\left( b-c \right)\left( c-a \right)$.

Lời giải của bài này trong cuốn sách như sau

Do vai trò của a, b, c có tính hoán vị vòng qubạn nên giả sử b nằm giữa a và c.

Nếu $a\ge b\ge c$ thì vế phải của BĐT âm, còn vế trái dương nên BĐT hiển nhiên.

Nếu $c\ge b\ge a$, ta có

$VP=4\left( a+b+c \right)\left( b-a \right)\left( c-b \right)\left( c-a \right)\le {{\left[ \left( a+b+c \right)\left( b-a \right)+\left( c-a \right)\left( c-b \right) \right]}^{2}}$

Ta chỉ cần chứng minh ${{a}^{2}}+{{b}^{2}}+{{c}^{2}}\ge \left( a+b+c \right)\left( b-a \right)+\left( c-b \right)\left( c-a \right)$. BĐT này tương đương với $a\left( 2a+2c-b \right)\ge 0$, đúng do $c\ge b\ge a$.

Mình xin hỏi:”Tại sao lại tách và sử dụng BĐT như vầy

$VP=4\left( a+b+c \right)\left( b-a \right)\left( c-b \right)\left( c-a \right)\le {{\left[ \left( a+b+c \right)\left( b-a \right)+\left( c-a \right)\left( c-b \right) \right]}^{2}}$

Tác giả đã dự đoán dấu bằng ra sao mà lại nhóm như thế? Đã sử dụng kỹ năng nào?

Mình đã thử làm khác đi là: giả sử $a=\max \left\{ a,b,c \right\}$. Nếu $b\ge c$ thì BĐT hiển nhiên. Xét $c\ge b$, sau đó mình làm như sau

$VP=4\left( a+b+c \right)\left( a-c \right)\left( c-b \right)\left( a-b \right)\le {{\left[ \left( a+b+c \right)\left( a-c \right)+\left( a-b \right)\left( c-b \right) \right]}^{2}}$

Ta chỉ cần chứng minh ${{a}^{2}}+{{b}^{2}}+{{c}^{2}}\ge \left( a+b+c \right)\left( a-c \right)+\left( a-b \right)\left( c-b \right)$. BĐT này tương đương với $c\left( 2c+2b-a \right)\ge 0$. Điều này là không chắc chắn.

Bạn hãy giải thích sai lầm của mình ở chỗ nào nhé.

Mình xin cảm ơn

 

 

 

- Thứ nhất: BĐT tác giả đã dùng là hệ quả của BĐT Cauchy 2 số: với x, y không âm (tức $\geq 0$) thì $4xy \leq x^2+y^2$ (*)

 

- Thứ hai: Mấy bài BĐT mà các biến tham gia có vai trò như nhau (hoán vị vòng quanh) thì dấu = xảy ra thường là các biến bằng nhau. Áp dụng đó để nhóm số cho phù hợp. Bạn có thể tham khảo kĩ thuật sử dụng BĐT Cauchy. (sách hoặc search)

 

- Thứ ba: cách khác của bạn chỉ là suy ra từ cách của tác giả, dĩ nhiên vẫn phải lựa chọn để nhóm các hạng tử mà sử dụng BĐT (*) kia vẫn đúng. Ở đây mình sửa theo cách của bạn là:

 

$VP=4\left( a+b+c \right)\left( a-c \right)\left( c-b \right)\left( a-b \right)\le {{\left[ \left( a+b+c \right)\left( c-b \right)+\left( a-b \right)\left(  a-c \right) \right]}^{2}}$

 

Ta chỉ cần chứng minh ${{a}^{2}}+{{b}^{2}}+{{c}^{2}}\ge {{\left[ \left( a+b+c \right)\left( c-b \right)+\left( a-b \right)\left(  a-c \right) \right]}^{2}}$. BĐT này tương đương với $b\left( 2c+2a-c \right)\ge 0$.

 

BĐT này đúng do $a \geq c \geq b$

 




#494878 $\sqrt[3]{17\sqrt{5}-38}$

Đã gửi bởi angleofdarkness on 24-04-2014 - 14:37 trong Đại số

Đây là HPT đối xứng loại II . Đặt a=tb ta có:

$\left\{\begin{matrix} b^{3}(5t^{3}+1)=17 & & \\ b^{3}(15t^{2}+1)=-38& & \end{matrix}\right.$

Chia theo vế 2 pt và thu được : $190t^{3}+105t^{2}+38t+1=0$

Bấm máy ra nghiệm.Nhưng ngiệm vô tỉ @@

P/s : @Hamhoctoan: cái dạng số phức tạp như thế này mình ko cần phân tích ra mũ 3 đâu.

Chắc nó phải có cái gì đặc biết như lập phương lên chẳng hạn.

 

 

Đặt $17\sqrt{5}-38=(a\sqrt{5}+b)^{3}\Leftrightarrow a^{3}5\sqrt{5}+15a^{2}b+ab^{2}3\sqrt{5}+b^{3}=17\sqrt{5}-38\Leftrightarrow \sqrt{5}(5a^{3}+ab^{2})+(15a^{2}b+b^{3})=17\sqrt{5}-38\Rightarrow \left\{\begin{matrix} 5a^{3}+ab^{2} & =17\\ 15a^{2}b+b^{3} & =-38 \end{matrix}\right.$ Sau đó GHPT là ra

P/s: HPT đó mình chưa giải được bạn thông cảm

 

Mấy bài phân tích căn bậc ba thế này thường trong căn sẽ phân tích thành một lập phương của một tổng, tổng này thường có một hạng tử là một số căn, ví dụ với bài này thì hạng tử đó là $\sqrt{5}$, bằng phép phân tích từ đó ta biến đổi như sau:

 

$\sqrt[3]{17\sqrt{5}+38}=\sqrt[3]{(\sqrt{5})^3+3.(\sqrt{5})^2.2+3.\sqrt{5}.2^2+2^3} \\ =\sqrt[3]{(\sqrt{5}+2)^3} \\ =\sqrt{5}+2$




#494692 1 số tính chất của tập hợp

Đã gửi bởi angleofdarkness on 23-04-2014 - 11:53 trong Đại số

tập hợp có tính chất $\left( {A \cup B} \right)\backslash C = \left( {A \cup C} \right)\backslash \left( {A \cup B} \right)$ và $\left( {A \cap B} \right)\backslash C = \left( {A \cap C} \right)\backslash \left( {A \cap B} \right)$ ko

 

Mình đoán là bạn hỏi để làm bài tập hợp trên TTT2 số vừa rồi đúng k?




#494691 Tìm nghiệm nguyên của phương trình:$x^{2}+x+6=y^{2}...

Đã gửi bởi angleofdarkness on 23-04-2014 - 11:51 trong Đại số

Tìm nghiệm nguyên của phương trình:$x^{2}+x+6=y^{2}$

 

Đưa về pt tích: $$x^{2}+x+6=y^{2} \\ \Leftrightarrow 4y^{2}-(4x^{2}+4x+1)=23 \\ \Leftrightarrow (2y)^{2}-(2x+1)^2=23 \\ \Leftrightarrow (2y-2x-1)(2y+2x+1)=23$$

 

Đến đây bạn tự làm tiếp. 




#494059 Tìm GTLN của $ A=\frac{2(x_{1}+x_{2})+7...

Đã gửi bởi angleofdarkness on 20-04-2014 - 10:26 trong Đại số

Giải:

b) Để pt có 2 nghiệm đều nguyên $\Leftrightarrow \Delta '=m^2+16$ là số chính phương $\Rightarrow$ $m \in Z^+$

Đặt $m^2+16=a^2(a \in N*)$ $\Leftrightarrow (a-m))(a+m)=16=2^2.2^2=2.2^3$

Vì $a-m\leq a+m\Rightarrow \left\{\begin{matrix} a-m=2^2 & & \\ a+m=2^2 & & \end{matrix}\right.$ hoặc $\left\{\begin{matrix} a-m=2 & & \\ a+m=2^3 & & \end{matrix}\right.$

Từ đó tìm được $m=0;m=3$ (tmđk)

P/s: Hình như câu b sai rồi!!!

 

Đúng rồi mà bạn, chỉ tội giải thích rõ ra là a - m và a + m cùng tính chắn lẻ.




#494054 Tìm nghiệm nguyên dương x,y,z biết $x^{2}+y^{2}+z^...

Đã gửi bởi angleofdarkness on 20-04-2014 - 10:24 trong Đại số

Tìm nghiệm nguyên dương x,y,z biết $x^{2}+y^{2}+z^{2}=xy+3y+2z-4$

 

$x^{2}+y^{2}+z^{2}=xy+3y+2z-4 \\ \Leftrightarrow 4x^{2}+4y^{2}+4z^{2}-4xy-12y-8z-16=0 \\ \Leftrightarrow (4x^{2}-4xy+y^2)+(3y^2-12y+12)+(4z^2-8z+4)=0 \\ \Leftrightarrow (2x-y)^2+3(y-2)^2+4(z-1)^2=0$

 

Như vậy có $(x;y;z)=(1;2;1)$




#494043 $\sum \frac{a^2}{b^2+c^2}\geq \s...

Đã gửi bởi angleofdarkness on 20-04-2014 - 09:56 trong Bất đẳng thức và cực trị

Cho các số thực dương a,b,c. Chứng Minh Rằng;

$\sum \frac{a^2}{b^2+c^2}\geq \sum \frac{a}{b+c}$

 

Đã có tại pic này 




#493956 Tính chiều dài quãng đường AB, biết rằng vận tốc của hai người không đổi tron...

Đã gửi bởi angleofdarkness on 19-04-2014 - 20:50 trong Các dạng toán khác

Gọi độ dài quãng đường AB là $x$ (km) (x > 0)
 
Gọi thời gian từ lúc xuất phát đến lúc gặp nhau lần 1 là $t$ (h)
 
$\Rightarrow$ Vận tốc của người xuất phát từ A là: $\dfrac{8}{t}$
 
Vận tốc của người xuất phát từ B là: $\dfrac{x-8}{t}$
 
Thời gian người đi từ A đi đến lúc gặp nhau lần thứ 2 là: $\dfrac{x+5}{\dfrac{8}{t}}=\dfrac{t(x+5)}{8}$
 
Thời gian người đi từ B đi đến lúc gặp nhau lần thứ hai là $\dfrac{2x-5}{\dfrac{x-8}{t}}=\dfrac{t(2x-5)}{x-8}$
 
$\Rightarrow \dfrac{t(x+5)}{8}=\dfrac{t(2x-5)}{x-8} \Leftrightarrow \dfrac{x+5}{8}=\dfrac{2x-5}{x-8}$
 
Đến đây giải ra là được.
 



#493952 $\left\{\begin{matrix} x^{3}-8=y...

Đã gửi bởi angleofdarkness on 19-04-2014 - 20:29 trong Phương trình, hệ phương trình và bất phương trình

 

giải hệ phương trình sau:

$x^{3}-8x=y^{3}+2y$

và $x^{3}-3=3(y^{2}+1)$

 

 

Cách 1: đã có tại pic này 

 

Cách 2: 

 

$\left\{\begin{matrix} x^{3}-8x= y^{3}+2y & \\ x^{2}-3=3(y^{2}+1) \end{matrix}\right. \Leftrightarrow \left\{\begin{matrix} x^3- 8x= y(y^2+ 2) & \\ x^2 = 3(y^2+ 2) \end{matrix}\right.$
 
$\Leftrightarrow \left\{\begin{matrix} x^3- 8x= y(y^2+ 2) & \\ \dfrac{x^2y}{3} = y(y^2+ 2) \end{matrix}\right.$
 

$\Rightarrow x^3 - 8x = \dfrac{x^2y}{3}$

 

Do x = 0 k là nghiệm của hệ nên $y=3(x - \dfrac{8}{x})$

 

Đến đây dùng cách thế.

@Trang Luong : Đề khác, pt $(1)$ là 8 chứ không phải $8x$

 

@angle: vậy tiêu đề khác với nd hỏi bài à -_-




#493945 CMR: Trong 3 phương trình sau có ít nhất 1 phương trình có nghiệm:...

Đã gửi bởi angleofdarkness on 19-04-2014 - 20:05 trong Đại số

CMR:
Trong 3 phương trình sau có ít nhất 1 phương trình có nghiệm:
$ax^2+2bx+c=0$
$bx^2+2cx+a=0$
$cx^2+2ax+b=0$

 

Cộng các biệt thức lại:

 

$P=\Delta_1 '+\Delta_2 '+\Delta_3 '=\sum (b^2-ab)=\sum a^2 - \sum ab$

 

Theo BĐT AM - GM thì $\sum a^2 \geq \sum ab$ nên có $P \geq 0$

 

Như vậy 1 trong 3 số $\Delta_1 ';\Delta_2 '$ và $\Delta_3 '$ sẽ có ít nhất 1 số dương.

 

Tức trong 3 phương trình cho có ít nhất 1 phương trình có nghiệm.




#493925 [Vio9](1;b) là nghiệm của phương trình 2x-3y=8 khi b=....

Đã gửi bởi angleofdarkness on 19-04-2014 - 19:18 trong Tài liệu - Đề thi

chỉ cần đáp án hay giải tắt ra luôn hả bạn?




#493917 Tìm x $\epsilon $ $\mathbb{Z}$ để A...

Đã gửi bởi angleofdarkness on 19-04-2014 - 19:02 trong Đại số

c/

 

* Nếu 4 < x < 8 (1) thì $A=\dfrac{4x}{x-4}=4+\dfrac{16}{x-4}$

 

Như vậy A nguyên khi x - 4 là ước của 16.

 

Kết hợp đk (1) suy ra 0 < x - 4 < 4 nên $x \in$ {1; 2}

 

Tính ra x = {5; 6}

 

* Nếu $x \geq 8$ thì $A=\dfrac{2x}{\sqrt{x-4}}$

 

Dễ c/m x - 4 là scp nên $x-4=m^2$ tức là $x=m^2+4$ (m tự nhiên)

 

Lúc đó biến đổi $A=2m+\dfrac{8}{m}$

 

Như vậy $m \in$ {1; 2; 4; 8}

 

Tính ea chọn $x \in$ {8; 20; 68}

 



#493916 Tìm x $\epsilon $ $\mathbb{Z}$ để A...

Đã gửi bởi angleofdarkness on 19-04-2014 - 18:54 trong Đại số

 

Bài 2: Cho $A= \frac{\sqrt{x + 4\sqrt{x + 4}} + \sqrt{x + 4\sqrt{x - 4}}}{\sqrt{\frac{16}{x^2} - \frac{8}{x} + 1}}$

a) Rút gọn A

b) Tìm GTNN của A

c) Tìm x $\epsilon $ $\mathbb{Z}$ để A $\epsilon $ $\mathbb{Z}$ 

 

 

a/

 

Bạn xem lại chỗ $\sqrt{x + 4\sqrt{x + 4}}$, trong căn phải là x - 4 chứ k phải x + 4 (nếu như bạn thì làm k ra  :D)

 

ĐKXĐ: với mọi  $x \in R;x>4$

$A= \dfrac{\sqrt{x + 4\sqrt{x - 4}} + \sqrt{x + 4\sqrt{x - 4}}}{\sqrt{\dfrac{16}{x^2} - \dfrac{8}{x} + 1}} \\ = \dfrac{\sqrt{x-4 + 4\sqrt{x - 4}+4} + \sqrt{x-4 + 4\sqrt{x - 4}+4}}{\sqrt{\Big( \dfrac{4}{x} - 1 \Big) ^2}} \\ = \dfrac{\sqrt{(\sqrt{x - 4}+2)^2} + \sqrt{(\sqrt{x - 4}-2)^2}}{|  \dfrac{4}{x} - 1|} \\ = \dfrac{\sqrt{x - 4}+2 + |\sqrt{x - 4}-2|}{|  \dfrac{4}{x} - 1|}$

 

Nếu 4 < x < 8 thì $A=\dfrac{4x}{x-4}$

 

Nếu $x \geq 8$ thì $A=\dfrac{2x\sqrt{x-4}}{x-4}$

 

b/

 

Nếu 4 < x < 8 thì $A=\dfrac{4x}{x-4}$ như vậy có A > 8

 

Nếu $x \geq 8$ thì $A=\dfrac{2x\sqrt{x-4}}{x-4}$

 
Dùng Cô si đc $A \geq 8$
 
Như vậy min A = 8 khi .... x = 8



#493910 Trên mặt phẳng tọa độ Oxy cho parabol (P) $y= x^{2}$ và đ...

Đã gửi bởi angleofdarkness on 19-04-2014 - 18:25 trong Đại số

Trên mặt phẳng tọa độ Oxy cho parabol (P) $y= x^{2}$ và đường thẳng (d) $y= mx+2$.

a) CMR với mọi giá trị của m thì đường thẳng (d) luôn cắt parabol (P) tại hai điểm nằm về hai phía trục tung.

b) Giả sử đường thẳng (d) cắt parabol (P) tại $A(x_{1};y_{1})$ và $B(x_{2};y_{2})$. Tìm giá trị của m để $\left | y_{1} -y_{2}\right |=\sqrt{24-x_{2}^{2}-mx_{1}}$

 

a/

 

Pt hoành độ giao điểm: $x^2=mx+2 \Leftrightarrow x^2-mx-2=0 (1)$

 

Xét 1 . (-2) = -2 < 0 nên (1) luôn có hai nghiệm $x_1;x_2$ trái dấu.

 

Tức là với mọi giá trị của m thì đường thẳng (d) luôn cắt parabol (P) tại hai điểm nằm về hai phía của trục tung.




#493907 Cho phương trình $x^{4}-(3m+14)x^{2}+(4m+12)(2-m)=0...

Đã gửi bởi angleofdarkness on 19-04-2014 - 18:09 trong Đại số

Cho phương trình $x^{4}-(3m+14)x^{2}+(4m+12)(2-m)=0 (1)$ có 4 nghiệm phân biệt. Tính GTLN của tích 4 nghiệm đó.

 

Đặt $y=x^2$ để (1) trở thành pt bậc hai ẩn y.

 

Bạn xét $\Delta$ để tìm đk của m sao cho pt ẩn y mới có 2 nghiệm y phân biệt dương để (1) có 4 nghiệm phân biệt x.

 

Lúc đó có P là tích 4 nghiệm phân biệt của (1) chính là tích 2 nghiệm y dương phân biệt của pt ẩn y mới.

 

Như vậy $P=y_1y_2=(4m+12)(2-m)=-4m^2-4m+24=-(2m+1)^2+25 \leq 25$

 

Đến đây chắc ra rồi.




#493901 MIN $T_{1}$+$T_{2}$+$T_{3...

Đã gửi bởi angleofdarkness on 19-04-2014 - 17:42 trong Bất đẳng thức và cực trị

Đây là bài trong đề thi vào lớp 10 THPT chuyên Bắc Ninh tỉnh mình năm kia, đã có ở topic này




#492847 $\frac{x+y\sqrt{2013}}{y+z\sqrt...

Đã gửi bởi angleofdarkness on 14-04-2014 - 12:10 trong Số học

Tìm x,y,z nguyên dương sao cho $\frac{x+y\sqrt{2013}}{y+z\sqrt{2013}}$ là số hữu tỉ,đồng thời x2+y2+z2 là số nguyên tố.

 

Đây là đề thi vào lớp 10 THPT chuyên Tỉnh Bắc Ninh của mình (vòng 2) năm ngoái.